Page 1 of 1

PT 17 Sec 3 #12

Posted: Thu May 03, 2012 5:32 pm
by meandme
Would some please explain this question and answer.

Thank you
God bless

Re: PT 17 Sec 3 #12

Posted: Thu May 03, 2012 6:31 pm
by Nova
Sure.

The arguement's conclusion is that the ad campaign directly decreased the amount of people who smoke. This is flawed, because there is not enough evidence to prove that the ad campaign caused the decrease, rather than the additional tax. This flaw is identifieable before getting to the anwer choices. The correct answer will strengthen the conclusion that the ad campaign affected the decrease in the amount of smokers.

The correct answer choice, (D), eliminates the possibility that the tax is the reason for the decrease because merchants slashed prices to offset it. Eliminating alternatives is common way to strengthen an arguement.

Hope that helps.

Re: PT 17 Sec 3 #12

Posted: Thu May 03, 2012 6:38 pm
by meandme
Thank you Nova. I understand now.